1977 USAMO Problems/Problem 5

Revision as of 15:16, 17 September 2012 by 1=2 (talk | contribs) (Problem)

Problem

If $a,b,c,d,e$ are positive numbers bounded by $p$ and $q$, i.e, if they lie in $[p,q], 0 < p$, prove that

\[(a+b +c +d +e)\left(\frac{1}{a} +\frac {1}{b} +\frac{1}{c} + \frac{1}{d} +\frac{1}{e}\right) \le 25 + 6\left(\sqrt{\frac {p}{q}} \minus{} \sqrt {\frac{q}{p}}\right)^2\] (Error compiling LaTeX. Unknown error_msg)

and determine when there is equality.

Solution

This problem needs a solution. If you have a solution for it, please help us out by adding it.

See Also

1977 USAMO (ProblemsResources)
Preceded by
Problem 4
Followed by
Last Question
1 2 3 4 5
All USAMO Problems and Solutions